Polynomial Again?

Algebra Level 5

{ x 3 x y z = 2 y 3 x y z = 6 z 3 x y z = 20 \begin{cases} x^3-xyz=2 \\ y^3-xyz =6 \\ z^3-xyz = 20 \end{cases}

If x , y x,y and z z are real solutions satisfying the system of equations above, and the maximum value of x 3 + y 3 + z 3 x^3+y^3+z^3 is equal to m n \dfrac mn , where m m and n n are coprime positive integers, find m + n m+n .


The answer is 158.

This section requires Javascript.
You are seeing this because something didn't load right. We suggest you, (a) try refreshing the page, (b) enabling javascript if it is disabled on your browser and, finally, (c) loading the non-javascript version of this page . We're sorry about the hassle.

2 solutions

Actually there are two solutions! And both of them are correct, it means that they hold all of given conditions. Here's my solution:

Those equations can be written as x 3 = 2 + x y z y 3 = 6 + x y z z 3 = 20 + x y z \begin{aligned} x^3 & = 2+xyz \\ y^3 & = 6+xyz \\ z^3 & = 20+xyz \end{aligned} Let A = x y z A=xyz . Multiply all of equations written above, we have A 3 = ( 2 + A ) ( 6 + A ) ( 20 + A ) , A^3=(2+A)(6+A)(20+A), which is finally a quadratic equation, whose solutions are A = 4 , 15 7 A=-4,-\frac{15}{7} . Instead of multiplying, if we sum them up, we have x 3 + y 3 + z 3 = 28 + 3 A x^3+y^3+z^3=28+3A , which can be either 16 16 or 151 7 \frac{151}{7} . Therefore, the answers are 17 17 and 158 158 .

To ensure that both solutions satisfy all condition, if we have A = 4 A=-4 , then x 3 = 2 + A = 2 x^3=2+A=-2 , and we get x = 2 3 x=-\sqrt[3]{2} . Similarly, we get y = 2 3 y=\sqrt[3]{2} , and z = 16 3 z=\sqrt[3]{16} . We see that x y z = 4 xyz=-4 , so, everything works fine. Moreover, if A = 15 7 A=-\frac{15}{7} , we have x = 1 7 3 x=-\sqrt[3]{\frac{1}{7}} , y = 27 7 3 y=\sqrt[3]{\frac{27}{7}} , z = 125 7 3 z=\sqrt[3]{\frac{125}{7}} . As a result, x y z = 15 7 xyz=-\frac{15}{7} . This also holds.

Edit: if we want the greatest x 3 + y 3 + z 3 x^3+y^3+z^3 , the answer is 158 158 .

I am sorry I was pretty busy yesterday, I have changed the question now, please check it.

Department 8 - 5 years, 5 months ago

Log in to reply

Lovely question!

Calvin Lin Staff - 5 years, 5 months ago

Nice inspiration from "mathematical olympiad challenges" by Titu Andreescu.!!

That type of question is there in the book.

Priyanshu Mishra - 5 years, 5 months ago

Log in to reply

Wrote the book is nice

Java Ku - 5 years, 5 months ago

I solved it correctly, but didn't type the answer correctly lol

James Wilson - 3 years, 10 months ago

There are in fact two \color{#D61F06}{\text{two}} solutions to the problem.

{ x 3 x y z = 2 x 3 2 = x y z y 3 x y z = 6 y 3 6 = x y z z 3 x y z = 20 z 3 20 = x y z \begin{cases} x^3 - xyz = 2 & \Rightarrow x^3 - 2 = xyz \\ y^3 - xyz = 6 & \Rightarrow y^3 - 6 = xyz \\ z^3 - xyz = 20 & \Rightarrow z^3 - 20 = xyz \end{cases} .

x 3 2 = y 3 6 = z 3 20 y 3 = x 3 + 4 z 3 = x 3 + 18 \begin{aligned} \Rightarrow x^3 - 2 & = y^3 - 6 = z^3 - 20 \\ \Rightarrow y^3 & = x^3 + 4 \\ z^3 & = x^3 + 18 \end{aligned}

Therefore,

x 3 x y z = 2 x 3 x ( x 3 + 4 ) ( x 3 + 18 ) 3 = 2 x 3 2 = x ( x 3 + 4 ) ( x 3 + 18 ) 3 ( x 3 2 ) 3 = x 3 ( x 3 + 4 ) ( x 3 + 18 ) x 9 6 x 6 + 12 x 3 8 = x 9 + 22 x 6 + 72 x 3 28 x 6 + 60 x 3 + 8 = 0 7 x 6 + 15 x 3 + 2 = 0 ( 7 x 3 + 1 ) ( x 3 + 2 ) = 0 \begin{aligned} x^3 - xyz & = 2 \\ x^3 - x\sqrt[3]{(x^3+4)(x^3+18)} &= 2 \\ x^3 - 2 & = x \sqrt [3] {(x^3+4)(x^3+18)} \\ (x^3 - 2)^3 & = x^3 (x^3+4)(x^3+18) \\ x^9 - 6x^6 + 12x^3 - 8 & = x^9 + 22x^6 + 72x^3 \\ 28x^6 + 60x^3 + 8 & = 0 \\ 7x^6 + 15x^3 + 2 & = 0 \\ (7x^3 + 1)(x^3+2) & = 0 \end{aligned}

{ x 3 = 1 7 x 3 + y 3 + z 3 = 1 7 + 1 7 + 4 + 1 7 + 18 = 151 7 m + n = 158 x 3 = 2 x 3 + y 3 + z 3 = 2 2 + 4 2 + 18 = 16 m + n = 17 \Rightarrow \begin{cases} x^3 = -\frac{1}{7} & \Rightarrow x^3 + y^3 +z^3 = -\frac{1}{7} + -\frac{1}{7} + 4 + -\frac{1}{7} + 18 = \frac{151}{7} & \Rightarrow m + n = \boxed{158} \\ x^3 = -2 & \Rightarrow x^3 + y^3 +z^3 = -2 - 2 + 4 - 2 + 18 = 16 & \Rightarrow m + n = 17 \end{cases}

I am sorry, sir I was pretty busy yesterday, I have changed the question now, please check it.

Department 8 - 5 years, 5 months ago

0 pending reports

×

Problem Loading...

Note Loading...

Set Loading...